mathematics,set and function,exponent and logarithmic and trigonometric function

set, relation, function, set theory,set operation,domain,range, onto function, one one correspondence,exponent and logarithmic function,graph
সাধারণ গণিত:
সমস্যাঃ
$f(a)=\dfrac{1+a^3+a^6}{a^3}$ হলে প্রমাণ কর যে, $f\left(a^2\right)=f\left(a^{-2}\right)$

         সৃজনশীল প্রশ্নঃ
সার্বিক সেট $U=\{x: x \in N$ এবং $x$ জোড় সংখ্যা $<10\}$
$A=\{x: x \in N$ এবং $3 \leq x \leq 8\}$
$\mathrm{B}=\{\mathrm{x}: \mathrm{x}$ মৌলিক সংখ্যা) $\}$
$\mathrm{C}=\mathrm{A} \setminus \mathrm{B}$ এবং $f\left(\mathrm{x}^{\prime}\right)=\frac{1+\mathrm{x}^{3}+\mathrm{x}^{6}}{\mathrm{x}^{3}}$
ক. A এবং B সেটকে তালিকা পদ্ধতিতে প্রকাশ কর।
থ. দেখাও যে, $f\left(\mathrm{x}^{3}\right)=f\left(\mathrm{x}^{-3}\right)$.
গ. $(\mathrm{A} \cup \mathrm{B}) \times \mathrm{C}$ নির্ণয় করে এর ডোমেন এবং রেঞ্জ লেখ।


 সৃজনশীল প্রশ্নঃ 
$U=\left\{x\in \mathbb{Z}:x^2<5\right\}$ এর উপসেট সমূহ
$A=${$x: x,12$ এর প্রকৃত গুণনীয়ক}
$B=\left\{x: x^2-3x+2=0\right \}$
$C=${$x:x^2>15$ এবং $x^3<225$}
(ক) $C$ সেটকে তালিকা পদ্ধতিতে প্রকাশ কর।
(খ) দেখাও যে, $P((A\cap B)\cup C)$ এর উপাদান সংখ্যা $2^n$ কে সমর্থন করে।
(গ) প্রমাণ কর যে, $(A-B)\cup (B-A)\cup (A\cap B)=A\cup B$.
(ঘ) $A$ ও $B$ এর উপাদান সংখ্যা যথাক্রমে $m$ ও $n$ হলে প্রমাণ কর যে $A\times B$ এর উপাদান সংখ্যা $m\times n$.
(ঙ) দেখাও যে, $P(A)\cap P(B)=P(A\cap B)$
(চ) $P(B^{\prime} \setminus C^{\prime})$ নির্ণয় কর ।
(ছ) $B$ ও $C$ এর উপাদান সংখ্যা যথাক্রমে $m$ ও $n$ এবং $\left(\dfrac{x}{2}-y,m\right)=\left(n,x-\dfrac{y}{2}\right)$ হলে $(x,y)$ নির্ণয় কর।
                        সৃজনশীল প্রশ্নঃ
 $P=${$y∈N: y^3>25$ এবং $y^4<260$ } ,
 $Q=${$a∈Z: a^2>4$ এবং $a^3<125$ } এবং 
$R=${$x∈N:x^2>14$ এবং $x^3≤224$ } 
(ক) $Q$ কে তালিকা পদ্ধতিতে প্রকাশ কর।
(খ) $P×Q$ এবং $(P∪Q)\setminus R$ নির্ণয় কর।
(গ) $R×P$ এর $x≤y$ শর্তে একটি অন্বয় $(F)$ এর ডোমেন ও রেঞ্জ নির্ণয় কর।
                      সৃজনশীল প্রশ্নঃ
 $A=${$x∈N:x^2>15$ এবং $x^3≤225$ } ,
$B=\left\{x∈Z:x^3-13x^2+42x=0\right\}$ এবং
$C=${$x∈Z:4≤x≤7$ এবং $x$ জোড়সংখ্যা } 
(ক) $A$ কে তালিকা পদ্ধতিতে প্রকাশ কর।
(খ) দেখাও যে, $P(B)$ এর উপাদান সংখ্যা $2^{\log_2⁡8}$  এর সমান।
(গ) $A\setminus C$ এবং $(A∩B)×C$ নির্ণয় কর।

                    সৃজনশীল প্রশ্নঃ
$U=\left\{x∈\mathbb{Z}:x^2<25\right\}$ এর উপসেট
 $A=${$x∈\mathbb{N}:x^2>8$ এবং $x^3<126$} এবং
 $B=${$x\in\mathbb{Z}:x,4$ এর গুণিতক}
(ক) $U$ কে তালিকা পদ্ধতিতে প্রকাশ কর।
(খ) $B^\prime $ কে তিনটি উপায়ে সেট গঠন পদ্ধতিতে প্রকাশ কর।
(গ) প্রমাণ কর যে, $P(B\setminus A)≠P(B)\setminus P(A)$.
                    (ক) নং প্রশ্নের সমাধানঃ
$U=\left\{0,\pm 1, \pm 2 ,\pm 3, \pm 4,\right\}$
                    (খ) নং প্রশ্নের সমাধানঃ
$A=\left\{3,4\right\}$
$B=\left\{0,-4,4\right\}$
$B^\prime=\left\{\pm 1,\pm 2,\pm 3\right\}$
              $=\left\{x\in \mathbb{Z}: 1\le x^2 \le 9\right\}$
              $=\left\{x\in\mathbb{Z}:1\le |x|\le 3\right\}$
              $=${$x\in \mathbb{Z}:x^2\ge 1$ এবং $x^4\le 81$ }

                      সৃজনশীল প্রশ্নঃ
সময়ের ফাংশন নিম্নরূপঃ
 $y=f(t)=\dfrac{t^2}{1+t^2+t^4 }$
(ক) $f(t)$ এর হর কে উৎপাদকে বিশ্লষণ কর।
(খ) প্রমাণ কর যে, $f\left(\tfrac{1}{x^2}\right)=f\left(x^2\right).$
(গ) $\dfrac{1}{y}=4$ হলে $t^3+\dfrac{1}{t^3}$  এর মান নির্ণয় কর।
                   সৃজনশীল প্রশ্নঃ
$y=f(x)=\dfrac{13-11x}{11-13x}$
(ক) $a$ এর কোন মানের জন্য $f(-a)=-2$ হবে?
(খ) প্রমাণ কর যে $f\left(\tfrac{1}{y}\right)=\dfrac{1}{x}$
(গ) $f(y)=\dfrac{1}{3}$ হলে $x$ এর মান কত?
গাণিতিক সমস্যাঃ
$A=${$x∈N: x^3>25$ এবং $x^4<256$ } এবং $a∈A$ যেখানে 
$y=f(a)=\dfrac{3a+1}{3a-1}$ 
(ক) $A$ কে তালিকা পদ্ধতিতে প্রকাশ কর।
(খ) $y$ এর মান দ্বারা গঠিত সেট $B$ হলে $P(B∪A)$ নির্ণয় কর।
(গ) $\dfrac{1-f\left(\tfrac{1}{x^2}\right)}{f\left(\tfrac{1}{x^2}\right)+1}=\dfrac{1}{7}$ হলে $x$ এর মান নির্ণয় কর।

সৃজনশীল প্রশ্নঃ
$U=\left\{x∈N:x<7\right\}$ এর দুটি উপসেট
$A=\left\{x: x^3-5x^2+6x=0\right\}$ এবং
$B=${$x:x$ বিজোড় পূর্ণসংখ্য এবং $x<7$}
(ক) $A$ কে তালিকা পদ্ধতিতে প্রকাশ কর।
(খ) $P(A∪B)$ নির্ণয় করে দেখাও যে তা $2^n$ সূত্রকে মেনে চলে।
যেখানে $n$ হলো $A∪B$ এর উপাদান সংখ্যা।
সৃজনশীল প্রশ্নঃ 
$A=\left\{ x\in \mathbb{N} :x^{2}-5x+6=0\right\} \\ B=\left\{ x\in \mathbb{N} :2 <x <6\right\} $
$C=${$x\in\mathbb {N}:x$ জোড় সংখ্যা এবং $x^2>3$ এবং $x^3\le 216$}
(ক) $C$ তালিকা পদ্ধতিতে প্রকাশ কর 
(খ) $P(B)$ নির্ণয় করে দেখাও যে , এর উপাদান সংখ্যা $2^{n(B)}$ কে সমর্থন করে 
(গ) প্রমাণ কর যে,$ \left( A\backslash B\right) \cap \left( B\backslash A\right) = \left( A\cap B\right) \backslash \left( A\cup B\right)$
সৃজনশীল প্রশ্নঃ
$f_1 (x)=x^3-kx^2+17x-10,\; $$f_2 (x)=x^2-3x+2$ এবং $f_1 (3)=-4$
(ক) $k$ এর মান নির্ণয় কর।
(খ) $f_1 (x)=0$ এবং $f_2 (x)=0$ এর সমাধান সেট যথাক্রমে $A$ ও $B$ হলে $P(A\setminus B)$ নির্ণয় কর।
(গ) $f_1 (x)=0$ এর সমাধান সেটের মানগুলো দ্বারা গঠিত ত্রিভুজের কোণগুলোর মান নির্ণয় কর।
সৃজনশীল প্রশ্নঃ
$U=\left\{x∈\mathbb{N}:0≤x≤7\right\}$ এর উপসেট 
$A=${$x∈\mathbb{Z}:x^2>15$ এবং $x^3<255$} এবং
$B=\left\{x∈\mathbb{Z}:x^3-14x^2+42x=0\right\}$ 
(ক) $P(B)$ নির্ণয় কর।
(খ) $A∪B$ কে তালিকা ও সেট গঠন পদ্ধতিতে প্রকাশ কর।
(গ) প্রমাণ কর যে, $P(A)∪P(B)≠P(A∪B).$
সৃজনশীল প্রশ্নঃ
$A=${$x:x$ মৌলিক সংখ্যা } এবং 
$B=${$x:x$ জোড় স্বাভাবিক সংখ্যা} এবং 
$C=B\setminus A$ .
(ক) $B^c $ নির্ণয় কর। 
(খ)প্রমাণ কর যে,$ A\cup B=\left( A\backslash B\right) \cup \left( B\backslash A\right)\cup \left( A\cap B\right)$
(গ) প্রমাণ কর যে,$\left( A\cap B\right) \times C=\left( A\times C\right) \cap \left( B\times C\right)$
সৃজনশীল প্রশ্নঃ
$f(x)=x^3+6x^2+11x+6,$ $g(y)=y^3+ky^2-4y+8,g(-2)=0$
(ক) $ K $এর মান নির্ণয় কর। 
(খ) $ x $ কোন মানের জন্য $ f(x) = 0 $ হবে ?
(গ) $ f(x) = 0 $ এবং $ g(y)= 0$ এর সমাধান সেট যথাক্রমে $ A $ এবং $ B $ হলে $ P(A- B ) $ নির্ণয় কর ।
সৃজনশীল প্রশ্নঃ
$A=\left\{2,3,2,2,3,\cdots \cdots \right\}$ এবং 
$B=${$x\in \mathbb{Z}: x^3>25$ এবং $x^4<264$}
$C=\left\{\mp 3,\mp 4,\mp 5\right \}$
(ক) $C$ কে সেট গঠন পদ্ধতিতে প্রকাশ কর।
(খ) $A\times B$ নির্ণয় কর ।
(গ) $P(B\cap C)$ এবং $(A\cup B)\setminus C$ নির্ণয় কর।
গাণিতিক সমস্যাঃ
$A=${$x\in\mathbb{N}:x^2>7$ এবং $x^4\le 625$} কে তালিকা পদ্ধতিতে প্রকাশ কর।
গাণিতিক সমস্যাঃ
$f(x-4)=x^2+2x+3$ হলে $f(2)$ এর মান কত?
সমাধানঃ
$f(2)=f(6-4)=6^2+2×6+3$

সৃজনশীল প্রশ্নঃ
$S=${$(x,y):x\in A , y\in A$এবং $y=2x$}
যেখানে $A=\left\{z\in \mathbb{Z}: z^2\le 9\right\}$
$S$ অন্বয়ের ডোমেন $D$ এবং রেঞ্জ $R$ .
(ক) $A$ কে তালিকা পদ্ধতিতে প্রকাশ কর।
(খ) $S$ কে তালিকা পদ্ধতিতে প্রকাশ করে $S^{-1}$ এর ডোমেন ও রেঞ্জ নির্ণয় কর।
(গ) $S$ অন্বয়ের লেখচিত্র অঙ্কন করে $S$ ফাংশন কিনা তা নির্ধারণ কর।
(ঘ) $P(D\cap R)$ , $R\setminus D$ এবং $D\times R$ নির্ণয় কর।
সৃজনশীল প্রশ্নঃ
$A=\left\{x\in \mathbb{N}: x^2-5x+6=0\right\}$ ;
$B=\left\{x\in\mathbb{N}:2<x<6\right\} $
$C=${$x\in\mathbb{N}: x$জোড়সংখ্যা এবং $x^2>3$ এবং $x^3≤216$} 
(ক) $C$ কে তালিকা পদ্ধতিতে প্রকাশ কর।
(খ) $P(B)$ নির্ণয় করে দেখাও যে, $P(B)$ এর উপাদান সংখ্যা $ 2^{n(B)}$ কে সমর্থন করে ।
(গ)প্রমাণ কর যে,$(A\setminus B)∩(B\setminus  A)=(A\cap B)\setminus (A∪B)$
সৃজনশীল প্রশ্ন:
$U=\left\{x∈Z:0≤x≤4 \right\}$,
 $A=\left\{x∈Z:x^3-5x^2+6x=0\right\}  $ ,এবং
$B=${$x∈U:x^2≥1$  এবং  $x^4<90$} 
  (ক) $A$ কে তালিকা পদ্ধতিতে প্রকাশ কর।
  (খ) $P(A\cap B)$ এবং $A×B$ এর মান নির্ণয় কর ।
  (গ) প্রমাণ কর যে,$A'\cup B' =(A\cap B)'$
সৃজনশীল প্রশ্ন:
সার্বিক সেট $U=${$x:x∈N$ এবং $ x≤6$}এর দুটি উপসেট যথাক্রমে 
$A=${$x:x$ মৌলিক সংখ্যা},$B=${$x:x $জোড় সংখ্যা} এবং $C=B\setminus A$
  (ক) $B^c $ নির্ণয় কর।
  (খ) প্রমাণ কর যে $A∪B=(A\setminus  B)∪(A∩B)∪(B\setminus A)$
  (গ)প্রমাণ কর যে, $(A∩B)×C=(A×C)∩(B×C)$.
গাণিতিক সমস্যাঃ
$f\left( x\right) =x^{3}+ax^{2}+a^{2}x+a^{3}$ হলে $a$ কোন পূর্ণসাংখ্যিক মানের জন্য $f(-2)=-65$ হবে?
গাণিতিক প্রশ্নঃ
$f\left( x\right) =x^{2}+ax+a^{2}$ হলে $a$ কোন পূর্ণসাংখ্যিক মানের জন্য $f(-3)=19$ হবে?
গাণিতিক সমস্যা:
$y=f(x)=\dfrac{11x-9}{13x-11}$ হলে প্রমাণ কর যে,$f(x)=y$
প্রমাণঃ
$y=f(x)=\dfrac{11x-9}{13x-11}$ 
এখানে,
$y=\dfrac{11x-9}{13x-11}$
বা,$13xy-11y=11x-9$
বা,$13xy-11x=11y-9$
বা,$x(13y-11)=11y-9$
$\therefore x=\dfrac{11y-9}{13y-11}\cdots\cdots(i)$
এবং 
$f(x)=\dfrac{11x-9}{13x-11}$
বা,$f(y)=\dfrac{11y-9}{13y-11}$
$\therefore f(y)=x$  (প্রমানিত)         [$(i)$ হতে]
অনুরূপভাবে সমাধান করঃ
১.$y=f(x)=\dfrac{3x-4}{5x-3}$ হলে প্রমাণ কর যে,$f(x)=y$
২.$y=f(x)=\dfrac{5x-3}{4x-5}$ হলে প্রমাণ কর যে,$f(x)=y$
৩.$y=f(x)=\dfrac{7x-4}{5x-7}$ হলে প্রমাণ কর যে,$f(x)=y$
৪.$y=f(x)=\dfrac{8x-5}{7x-8}$ হলে প্রমাণ কর যে,$f(x)=y$
৫.$y=f(x)=\dfrac{10x-7}{5x-10}$ হলে প্রমাণ কর যে,$f(x)=y$
৬.$y=f(x)=\dfrac{15x-13}{4x-15}$ হলে প্রমাণ কর যে,$f(y)=x.$
৭.$y=f(x)=\dfrac{17x-4}{5x-17}$ হলে প্রমাণ কর যে $f(y)=x.$
৮.$y=f(x)=\dfrac{18x-5}{7x-18}$ হলে প্রমাণ কর যে,$f(y)=x.$
৯.$y=f(x)=\dfrac{15x-7}{5x-15}$ হলে প্রমাণ কর যে,$f(y)=x.$
১০.$y=f(x)=\dfrac{11x-13}{9x-11}$ হলে প্রমাণ কর যে $f(y)=x.$
১১.$y=f(x)=\dfrac{13x-4}{5x-13}$  হলে প্রমাণ কর যে,$f(y)=x.$
সৃজনশীল প্রশ্নঃ
 $y=f(x)=\dfrac{13-11x}{11-13x}$
(ক) $a$ এর কোন মানের জন্য $f(-a)=-2$ হবে?
(খ) প্রমাণ কর যে $f\left(\tfrac{1}{y}\right)=\dfrac{1}{x}.$
(গ) $f(y)=\dfrac{1}{3}$ হলে $x$ এর মান কত?
সৃজনশীল প্রশ্নঃ
$A=\left\{x\in\mathbb{N}: x^2-6x+5=0\right\}$
$B=\left\{x\in\mathbb{Z}:2<x<6\right\}$ 
$C=${$x\in\mathbb{N}: x$ বিজোড়সংখ্যা এবং $x^2>3$ এবং $x^3<216$} 
(ক) $C$ কে তালিকা পদ্ধতিতে প্রকাশ কর।
(খ) $P(B)$ নির্ণয় করে দেখাও যে,$P(B)$ এর উপাদান সংখ্যা $2^{n(B)}$ কে সমর্থন করে ।
(গ)প্রমাণ কর যে,$(A\setminus B)\cap (B-A)=(A\cap B)\setminus(A∪B)$
সৃজনশীল প্রশ্নঃ
$U=\left\{x∈Z:0≤x≤4 \right\}$,
 $A=\left\{x∈Z:x^3-5x^2+6x=0\right\} $ ,এবং
$B=${$x∈U:x^2≥1 $ এবং  $x^4<90$} 
  (ক) $A$ কে তালিকা পদ্ধতিতে প্রকাশ কর।
  (খ) $P(A∩B)$ এবং $A×B$ এর মান নির্ণয় কর ।
  (গ) প্রমাণ কর যে,$A^\prime ∪B^\prime=(A∩B)^\prime $ 
সৃজনশীল প্রশ্নঃ
সার্বিক সেট $U=${$x:x∈N$ এবং $x≤6$}এর দুটি উপসেট যথাক্রমে 
$A=${$x:x$ মৌলিক সংখ্যা}, 
$B=${$x:x$ জোড় সংখ্যা}এবং $C=B\setminus A$
(ক) $B^c$ নির্ণয় কর।
(খ) প্রমাণ কর যে $A∪B=(A\setminus B)∪(A\cap B)∪(B\setminus A)$
  (গ)প্রমাণ কর যে,$(A∩B)×C=(A×C)∩(B×C)$
সৃজনশীল প্রশ্নঃ
$A=\left\{x∈N:x^3-6x^2+11x-6=0\right\}$
 $B=\left\{x∈N:4<x^2<16\right\}$ এবং 
$C=\left\{x∈Z:x^2≤4\right\}$ 
(ক) $A$ সেটটিকে তালিকা পদ্ধতিতে প্রকাশ করো।
(খ)$P(A∪B)$ নির্ণয় কর এবং প্রমাণ করো যে, $P(A∪B)$ এর উপাদান সংখ্যা $2^{n(A∪B)}$  কে সমর্থন করে।
(গ) $F=${$(x,y):x∈C,y∈C$ এবং $x+y=1$} অন্বয়টিকে তালিকা পদ্ধতিতে প্রকাশ করে ডোমেন  ও রেঞ্জ নির্ণয় করো।
সৃজনশীল প্রশ্নঃ
 $f(x)=\dfrac{x-1}{x+1} $ ,$g(y)=\dfrac{y(y-1)}{3y^2-y^3-1}$ এবং $\left(ax-cy,f(2)\right)=\left(g(1),ay-cx\right)$
    (ক)$(x,y)$ নির্ণয় কর।
    (খ)প্রমাণ কর যে,$\dfrac{f(x)-f(y)}{1+f(x)f(y)}=\dfrac{x-y}{1+xy}$
    (গ)দেখাও যে,$g\left(\tfrac{1}{y^2}\right)=g\left(1-y^2\right)$
    (ঘ)$f\left(\tfrac{a}{b}\right)÷f\left(\tfrac{b}{a}\right)$ অথবা $g\left(\tfrac{p}{q}\right)÷g\left(\tfrac{q}{p}\right)$ নির্ণয় কর।
সৃজনশীল প্রশ্নঃ
$f(x)=x^3+6x^2+11x+6 ,$$g(y)=y^3+ky^2-4y+8  ,g(-2)=0$
  (ক) $k$ এর মান নির্ণয় কর ।
  (খ) $x$ এর কোন কোন মানের জন্য $f(x)=0$ হবে? 
  (গ) $g(y)=0$ হলে $y$ এর  মান নির্ণয় কর এবং $x∈A ,y∈B$ হলে $P(A-B)$ নির্ণয় কর ।
সৃজনশীল প্রশ্নঃ
$y=f(x)=\dfrac{7-5x}{5-3x}   ;  g(t)=\dfrac{2t+1}{2t-1}$  
এবং  $\left(\dfrac{a}{3}+\dfrac{b}{2},g(0)\right)=\left(f(1),\dfrac{b}{3}+\dfrac{a}{2}\right)$
(ক) প্রমাণ কর যে,$f(y)=x.$
(খ) $\dfrac{g(y)+1}{g(y)-1}$ নির্ণয় কর।
(গ)$\dfrac{g\left(\dfrac{1}{z^2}\right)+1}{g\left(\dfrac{1}{z^2}\right)-1}=\dfrac{1}{8}$ হলে $z$ এর মান নির্ণয় কর।
(ঘ) $(a,b)$ নির্ণয় কর।
সৃজনশীল প্রশ্নঃ
$R=${($x,y$)$:x∈A,y∈A$ এবং $x=y^2-2$} 
যেখানে $A=\left\{t∈Z:t^2<9\right\}$ এবং $R$ এর ডোমেন $P$ এবং রেঞ্জ $Q$.
(ক) অন্বয় ও ফাংশনের সংজ্ঞা লিখ।
(খ) $R$ কে তালিকা পদ্ধতিতে প্রকাশ কর।
(গ) প্রমাণ কর যে $\left(P∩Q\right)^\prime=P^\prime∪Q^\prime$.
(ঘ) $Q\setminus P ,P×Q$ এবং $P(Q)$ নির্ণয় কর।
(ঙ) $R$ এর ক্রমজোড়গুলো ছক বসিয়ে যে লেখচিত্র পাওয়া যায় তার ক্ষেত্রফল নির্ণয় কর।
সৃজনশীল প্রশ্নঃ
$y=f(x)=\dfrac{x(1-x)}{x^3-3x^3+1}$ এবং $A=\left\{0,1,3\right\}$
(ক) $x$ এর কোন কোন মানের জন্য $f(x)=0$.
(খ) প্রমাণ কর যে, $f\left(\tfrac{1}{y}\right)=f(1-y)$.
(গ) $R={(x,y):x∈A}$ এর ক্রোমজোড়গুলো ছক বসিয়ে যে চিত্র পাওয়া যায় তার ক্ষেত্রফল নির্ণয় কর।  
সৃজনশীল প্রশ্নঃ
$f (y)=\dfrac{y (1-y)}{y ^3-3y ^2+1}$
(ক)y এর কোন কোন মানের জন্য $f(y)=0$ হবে ?
(খ) প্রমাণ কর যে , $f\left(x^{-1}\right)=f (1-x).$
(গ)প্রমাণ কর যে  $f\left(\dfrac{1}{a ^2}\right)=f\left(1-a^2\right)$
(ঘ) প্রমাণ কর যে $f\left(\dfrac{1}{a ^3}\right)=f\left(1-a^3\right)$
(ঙ) প্রমাণ কর যে  $f\left(\dfrac{1}{a ^n}\right)=f\left(1-a^n\right)$
অনুরূপঃ



সৃজনশীল প্রশ্নঃ
 উদ্দীপকের আলোকে নিচের প্রশ্নগুলোর উত্তর দাওঃ
$131 , 311, 419$ সংখ্যাটিকে কিছুসংখ্যক স্বাভাবিক সংখ্যা দ্বারা ভাগ করলে $23$ অবশিষ্ট থাকে।
(ক) প্রথম সংখ্যাটির মৌলিক গুণনীয়ক সমূহের সেট নির্ণয় কর।
(খ)  প্রথম বা দ্বিতীয় সংখ্যাটি নিঃশেষে ভাগ করা যায় এমন সংখ্যা সমূহের সেট নির্ণয় কর।
(গ) সংখ্যা তিনটিকে ভাগ করা যায় এমন সংখ্যাসমূহের সেট নির্ণয় কর।
                        বহুনির্বাচনী প্রশ্ন:
                         মডেল টেষ্ট-১
পূর্ণমান-২৫                                        সময়-২৫ মিনিট
১.$\left\{x∈Z:9<x<10\right\}$সেটটি কি ধরনের ?
  (ক) সসীম সেট       (খ) অসীম সেট    (গ) নিশ্ছেদ সেট    (ঘ) ফাকা সেট 
২.$\left\{x∈N:9<x<10\right\}$সেটটি কি ধরনের ?
  (ক) সসীম সেট       (খ) অসীম সেট    (গ) নিশ্ছেদ সেট    (ঘ) ফাকা সেট 
৩.$\left\{x∈R:9<x<10\right\}$সেটটি কি ধরনের ?
  (ক) সসীম সেট       (খ) অসীম সেট    (গ) নিশ্ছেদ সেট    (ঘ) ফাকা সেট 
৪.$\left\{3,2,3,2,3,2,3,2,..........\right\}$সেটটি কি ধরনের ?
  (ক) সসীম সেট       (খ) অসীম সেট    (গ) নিশ্ছেদ সেট    (ঘ) ফাকা সেট 
৫.যদি $\left(2x-y,3\right)=\left(6,x-y\right)$ হয়,
তাহলে $(x, y)$ এর মান কোনটি?
ক. $(0,0)$     খ. $(3,0)$      গ. $(0,3)$     ঘ. $(3,3)$
৬. $A=\left\{x\in  N:x^3+5x^2+6x=0\right\}$ হলে $P(A)$ সেটের উপাদান সংখ্যা কত?
 (ক) $0$    (খ) $1$      (গ) $2$     (ঘ) $3$
৭.  $∅$ 
   $i.$ যেকোনো সেটের উপসেট 
   $ii.$ এর শক্তি সেটের উপাদান শূণ্য
   $iii.$ এর পূরক সেটই সার্বিক সেট 
    কোনটি সঠিক?
(ক) $i,ii$     (খ) $i,iii$     (গ) $ii,iii$     (ঘ) $i,ii,iii$
৮. নিচের কোন সম্পর্কটি সঠিক?
(ক) $\mathbb{N}⊂\mathbb{Z}⊂\mathbb{R}$                  (খ) $\mathbb{Q}⊂\mathbb{Z}⊂\mathbb{R}$
(গ) $\mathbb{Z}⊂\mathbb{N}⊂\mathbb{R}$                   (ঘ) $\mathbb{R}⊃\mathbb{Q}⊂\mathbb{Z}$
৯.নিচের গাণিতিক বাক্যগুলো লক্ষ্য কর-
$i. A∩B^\prime =${$x: x∈A$ অথবা $x∉B$ }
$ii. A^\prime =\left\{x: x∉A\right\}$
$iii. A×B=${$(x,y):x∈A$ এবং $y∈B$ }
কোনটি সঠিক?
(ক) $i,ii$     (খ) $i,iii$     (গ) $ii,iii$     (ঘ) $i,ii,iii$
নিচের তথ্যের ভিত্তিতে $১০-১২$ নং প্রশ্নের উত্তর দাওঃ
 $U=\left\{a,b,c\right\},\;X=\left\{a,b\right\},$$\;Y=\left\{b,c\right\}$   এবং $Z=\left\{c,a\right\}$
১১. $X∩Y∩Z^\prime $ এর উপাদান সংখ্যা কত?
(ক) $3$       (খ) $2$        (গ) $1$        (ঘ) $0$
১২. $P(Y∩Z∩X)=$  কত?
(ক) $\left\{∅\right\}$                             (খ) $\left\{\left\{a\right\},\left\{b\right\},\left\{c\right\},∅\right\}$     
(গ) $\left\{\left\{a\right\},\left\{c\right\},∅\right\}$               (ঘ) $\left\{ \right\}$
১৩. $\left(X∩Y\right)×Z=$ কত?
(ক) $\left\{(b,c),(a,c)\right\}$       (খ) $\left\{(b,c),(b,a)\right\}$    
 (গ) $\left\{(b,c),(a,b)\right\}$       (ঘ) $\left\{(a,b),(b,c),(c,a)\right\}$
১8.$A=${$x\in \mathbb{N}:x$ জোড় সংখ্যা } হলে $A^\prime =$ কত?
(ক) $\left\{2,4,6,8\right\}$                    (খ) $\left\{1,3,5,7\right\}$    
 (গ) $\left\{2,4,6,……\right\}$             (ঘ)$\left\{1,3,5,…….\right\}$
১৫. $f(x)=\dfrac{x+1}{x^2-2x+1}$ হলে $f(1)=$কত?
(ক) $0$           (খ) $2$         (গ) অসঙ্গায়িত        (ঘ) $\dfrac{2}{3}$
১৬. $M=${$x∈Z: x^2>14$ এবং $x^3<105$ } এর তালিকা পদ্ধতিতে প্রকাশ-
(ক) $\left\{………,-5,-4,4\right\}$      (খ) $\left\{±5,±4\right\}$   
 (গ) $\left\{……,-4,4,5\right\}$               (ঘ) $\left\{∓3,∓4,∓5\right\}$

                    উচ্চতর গণিত
গাণিতিক প্রশ্নঃ
$A\subseteq B$ হবে যদি এবং কেবল যদি নিম্নোক্ত যেকোন একটি শর্ত খাটে-
(ক) $A\cap B=A$
(খ) $A\cup B=B$
(গ) $B^\prime \subseteq A^\prime$
(ঘ) $A\cap B^\prime=\emptyset$
(ঙ) $A^\prime \cup B=U$
(ক) নং প্রশ্নের সমাধানঃ
$A\subseteq B$ হলে প্রমাণ করতে হবে যে, $A\cap B=A$
প্রমাণঃ
মনেকরি, 
      $x\in A\cap B$
$\Rightarrow x\in A$ এবং $x\in B$           

$\Rightarrow x\in A$                                         $[\because A\subseteq B]$
$\therefore A\cap B\subseteq A\cdots\cdots (i)$
মনেকরি, 
      $x\in A$
$\Rightarrow x\in A$ এবং $x\in B$                    $[\because A\subseteq B]$
$\Rightarrow x\in A\cap B$ 
$\therefore A\subseteq A\cap B \cdots\cdots (ii)$
$(i)$ ও $(ii)$ নং হতে পাই, $A\cap B=A$
আবার,
$A\cap B=A$ হলে প্রমাণ করতে হবে যে, $A\subseteq B.$
প্রমাণঃ
মনে করি,
      $x\in A$
$\Rightarrow x\in A\cap B$                                $[\because A\cap B=A]$
$\therefore A\subseteq A\cap B$
কিন্তু $A\cap B\subseteq B$
$\therefore A\subseteq B$
সুতরাং $A\subseteq B$ হবে যদি এবং কেবল যদি $A\cap B=A$ হয়।
(খ) নং প্রশ্নের সমাধানঃ
$A\subseteq B$ হলে প্রমাণ করতে হবে যে, $A\cup B=B$
প্রমাণঃ
মনেকরি, 
      $x\in A\cup B$
$\Rightarrow x\in A$ অথবা $x\in B$
$\Rightarrow x\in B$ অথবা $x\in B$            $[\because A\subseteq B]$
$\Rightarrow x\in B\cup B$
$\Rightarrow x\in B$
$\therefore A\cup B\subseteq B\cdots\cdots (i)$
কিন্তু $B\subseteq A\cup B\cdots\cdots (ii)$
$(i)$ ও $(ii)$ নং সমীকরণ হতে পাই, $A\cup B=B$
আবার, $A\cup B=B$ হলে প্রমাণ করতে হবে যে, $A\subseteq B$
মনে করি, 
      $x\in A$
$\Rightarrow x\in A\cap B$                 [$\because A\cup B=B$ হলে $A\cap B=A$]
$\therefore A\subseteq A\cap B$
কিন্তু $A\cap B\subseteq B$
$\therefore A\subseteq B$
(গ) নং প্রশ্নের সমাধানঃ
$A\subseteq B$ হলে প্রমাণ করতে হবে যে, $B^\prime \subseteq A^\prime$
প্রমাণঃ
মনে করি, 
      $x\in B^\prime $
$\Rightarrow x\notin B$
$\Rightarrow \notin A$                                      $[\because A\subseteq B]$
$\Rightarrow x\in A^\prime$
$\therefore B^\prime \subseteq A^\prime$
আবার, 
$B^\prime \subseteq A^\prime$ হলে প্রমাণ করতে হবে যে, $A\subseteq B$.
মনেকরি, 
      $x\in A$
$\Rightarrow x\notin A^\prime$
$\Rightarrow x\notin B^\prime$                   $[ B^\prime \subseteq A^\prime]$
$\Rightarrow x\in B$
$\therefore A\subseteq B$
(ঘ) নং প্রশ্নের সমাধানঃ
$A\subseteq B$ হলে প্রমাণ করতে হবে যে, $A\cap B^\prime=\emptyset $
মনেকরি, 
      $x\in A\cap B^\prime$
$\Rightarrow x\in A$ এবং $x\in B^\prime$
$\Rightarrow x\in B$ এবং $x\in B^\prime$         $[\because A\subseteq B]$
$\Rightarrow x\in B$ এবং $x\notin B$
$\Rightarrow x\in B\setminus B$
$\Rightarrow x\in \emptyset$
$\therefore A\cap B^\prime \subseteq \emptyset$
কিন্তু $\emptyset \subseteq A\cap B^\prime$
$\therefore A\cap B^\prime=\emptyset$
আবার, $A\cap B^\prime=\emptyset $ হলে প্রমাণ করতে হবে যে, $A\subseteq B$.
বিকল্প সমাধানঃ
মনেকরি, 
      $x\in A\cap B^\prime$
$\Rightarrow x\in A$ এবং $x\in B^\prime$
$\Rightarrow x\in (A\cap B)$ এবং $x\in B^\prime$          $[\because A\subseteq B\Leftrightarrow A\cap B=A]$
$\Rightarrow x\in A$ এবং $x\in B$ এবং $x\notin B$
$\Rightarrow x\in A$ এবং $x\in B\setminus B$
$\Rightarrow x\in A$ এবং $x\in \emptyset$
$\Rightarrow x\in A\cap \emptyset$
$\Rightarrow x\in \emptyset$
$\therefore A\cap B^\prime \subseteq \emptyset$   [উল্টোদিকে প্রমাণ করা যায়]
কিন্তু $\emptyset \subseteq A\cap B^\prime$
$\therefore A\cap B^\prime=\emptyset$
প্রমাণঃ 
মনে করি, 
      $x\in A$
$\Rightarrow x\notin B^\prime$            $[\because A\cap B^\prime =\emptyset]$   
$\Rightarrow x\in \left(B^\prime\right)^\prime$
$\Rightarrow x\in B$ 
$\therefore A\subseteq B$
(ঙ) নং প্রশ্নের সমাধানঃ
$A\subseteq B$ হলে প্রমাণ করতে হবে যে, $B\cup A^\prime =U$.
মনেকরি,
      $x\in B\cup A^\prime$
$\Rightarrow x\in B$ অথবা $x\in A^\prime$
$\Rightarrow x\in (A\cup B)$ অথবা $x\in A^\prime$         $\left[\because A\subseteq B \Leftrightarrow A\cup B=B\right]$
$\Rightarrow x\in A$ অথবা $x\in B$ অথবা $x\in  A^\prime$
$\Rightarrow x\in A $ অথবা $x\in A^\prime$ অথবা $x\in B$
$\Rightarrow x\in A\cup A^\prime$ অথবা $x\in B$
$\Rightarrow x\in U$ অথবা $x\in B$
$\Rightarrow x\in U\cup B$
$\Rightarrow x\in U$
$\therefore B\cup A^\prime \subseteq U\cdots\cdots (i)$
মনেকরি,
      $x\in U$ 
$\Rightarrow x\in U\cup B$
$\Rightarrow x\in U$ অথবা $x\in B$
$\Rightarrow x\in A\cup A^\prime$ অথবা $x\in B$
$\Rightarrow x\in A $ অথবা $x\in A^\prime$ অথবা $x\in B$
$\Rightarrow x\in A$ অথবা $x\in B$ অথবা $x\in A^\prime$
$\Rightarrow x\in (A\cup B)$ অথবা $x\in A^\prime$ $\left[\because A\subseteq B \Leftrightarrow A\cup B=B\right]$
$\Rightarrow x\in B$ অথবা $x\in A^\prime$
$x\in B\cup A^\prime$
$\therefore U\subseteq B\cup A^\prime\cdots\cdots (ii)$
$(i)$ এবং $(ii)$ নং সমীকরণ হতে পাই,
 $B\cup A^\prime =U$
বিকল্পঃ
মনেকরি, 
    $B\cup A^\prime $
$\Rightarrow x\in B$ অথবা $x\in A^\prime $
$\Rightarrow x\in B$ অথবা $x\in B^\prime \cup A^\prime$  $\left[\because A\subseteq B \Leftrightarrow B^\prime \subseteq A^\prime \Leftrightarrow B^\prime\cup A^\prime=A^\prime\right]$
$\Rightarrow x\in B$ অথবা $x\in B^\prime $ অথবা $x\in A^\prime$
$\Rightarrow x\in B\cup B^\prime$ অথবা $x\in A^\prime$
$\Rightarrow x\in U$ অথবা $x\in A^\prime$
$\Rightarrow x\in U\cup A^\prime$
$\Rightarrow x\in U$
$\therefore B\cup A^\prime\subseteq U$
মনেকরি, 
      $x\in U$
$\Rightarrow x\in U\cup A^\prime$
$\Rightarrow x\in U$ অথবা $x\in A^\prime$
$\Rightarrow x\in B\cup B^\prime$ অথবা $x\in A^\prime$
$\Rightarrow x\in B$ অথবা $x\in B^\prime $ অথবা $x\in A^\prime$
$\Rightarrow x\in B$ অথবা $x\in B^\prime \cup A^\prime$    
$\Rightarrow x\in B$ অথবা $x\in A^\prime $     $\left[\because A\subseteq B \Leftrightarrow B^\prime \subseteq A^\prime \Leftrightarrow B^\prime\cup A^\prime=A^\prime\right]$
$\Rightarrow x\in B\cup A^\prime$
$\therefore U\subseteq B\cup A^\prime $
$\therefore B\cup A^\prime =U$
$B\cup A^\prime =U$ হলে প্রমাণ করতে হবে যে, $A\subseteq B$
মনেকরি, 
      $x\in A$
$\Rightarrow x\notin A^\prime $
$\Rightarrow x\in \left(B\setminus A^\prime\right)$      $\left[\because B\cup A^\prime =U \right]$
$\Rightarrow x\in B$ এবং $x\notin A^\prime$
$\Rightarrow x\in B$ এবং $x\in A$
$\Rightarrow x\in B\cap A$
$\Rightarrow x\in A\cap B$    [বিনিময় বিধি]
$\therefore A\subseteq A\cap B$
কিন্তু $A\cap B \subseteq B$
$\therefore A\subseteq B$
গাণিতিক প্রশ্নঃ
প্রমাণ কর যে,
(ক) $A\setminus B\subseteq A\cup B$
(খ) $A^\prime\setminus B^\prime=B\setminus A$
(গ) $A\setminus B\subseteq A$
(ঘ) $A\subseteq B$ হলে $A\cup \left(B\setminus A\right)=B$
(ঙ) $A\cap B=\emptyset $ হলে প্রমাণ কর যে, $A\subseteq B^\prime$ এবং $A\cap B^\prime=A$ এবং $A\cup B^\prime=B^\prime$
(ক)নং প্রশ্নের সমাধানঃ
মনেকরি,
      $x\in A\setminus B$
$\Rightarrow x\in A$ এবং $x\notin B$
$\Rightarrow x\in A$
$\therefore A\setminus B\subseteq A$
কিন্তু $A\subseteq A\cup B$
সুতরাং $A\setminus B \subseteq A\cup B$
(খ) নং প্রশ্নের সমাধানঃ
মনেকরি, 
      $x\in A^\prime \setminus B^\prime $
$\Rightarrow x\in A^\prime $ এবং $x\notin B^\prime$
$\Rightarrow x\notin A$ এবং $x\in B$
$\Rightarrow x\in B$ এবং $x\notin A$
$\Rightarrow x\in B\setminus A$
$\therefore A^\prime \setminus B^\prime\subseteq B\setminus A$
মনেকরি,
$\Rightarrow x\in B\setminus A$
$\Rightarrow x\in B$ এবং $x\notin A$
$\Rightarrow x\notin A$ এবং $x\in B$
$\Rightarrow x\in A^\prime $ এবং $x\notin B^\prime$
$\Rightarrow x\in A^\prime \setminus B^\prime $
$\therefore B\setminus A\subseteq A^\prime \setminus B^\prime$
$A^\prime \setminus B^\prime= B\setminus A$
(গ) নং প্রশ্নের সমাধানঃ
মনেকরি,
      $x\in A\setminus B$
$\Rightarrow x\in A$ এবং $x\notin B$
$\Rightarrow x\in A$
$\therefore A\setminus B\subseteq A$
(ঘ) নং প্রশ্নের সমাধানঃ
 মনেকরি, 
      $x\in A\cup \left(B\setminus A\right)$
$\Rightarrow x\in A$ অথবা $x\in \left(B\setminus A\right)$
$\Rightarrow x\in A$ অথবা ($x\in B$ এবং $x\notin A$)
$\Rightarrow$ ($x\in A$ অথবা $x\in B$) এবং ($x\in A$ অথবা $x\in A^\prime$)
$\Rightarrow$ ($x\in A$ অথবা $x\in B$) এবং $x\in\left(A\cup A^\prime\right)$
$\Rightarrow x\in A\cup B$ এবং $x\in U$
$\Rightarrow x\in B$ এবং $x\in U$     $[\because A\subseteq B\Leftrightarrow A\cup B=B]$
$\Rightarrow x\in (B\cap U)$
$\Rightarrow x\in B$
$\therefore A\cup \left(B\setminus A\right)\subseteq B$
মনেকরি,
      $x\in B$
$\Rightarrow x\in (B\cap U)$
$\Rightarrow x\in B$ এবং $x\in U$ $[\because A\subseteq B\Leftrightarrow A\cup B=B]$
$\Rightarrow x\in A\cup B$ এবং $x\in U$
$\Rightarrow (x\in A$ অথবা $x\in B$) এবং $x\in\left(A\cup A^\prime\right)$
$\Rightarrow$ ($x\in A$ অথবা $x\in B$) এবং ($x\in A$ অথবা $x\in A^\prime$)
$\Rightarrow x\in A$ অথবা ($x\in B$ এবং $x\notin A$)
$\Rightarrow x\in A$ অথবা $x\in \left(B\setminus A\right)$
$\Rightarrow x\in A\cup \left(B\setminus A\right)$
$\therefore B\subseteq A\cup \left(B\setminus A\right)$
$\therefore A\cup \left(B\setminus A\right)= B$

(ঙ) নং প্রশ্নের সমাধানঃ
মনেকরি, 
      $x\in A$
$\Rightarrow x\notin B $    $[\because A\cap B=\emptyset]$
$\Rightarrow x\in B^\prime$
$\therefore A\subseteq B^\prime$  (প্রমাণিত)
মনেকরি,
      $x\in A\cap B^\prime$
$\Rightarrow x\in A $ এবং $ x\in B^\prime$
$\Rightarrow x\in A$ এবং $x\notin B$
$\Rightarrow x\in A$ এবং $x\in A$
$\Rightarrow x\in A\cap A$
$\Rightarrow x\in A$
$\therefore A\cap B^\prime\subseteq A$
মনেকরি,
      $x\in A$
$\Rightarrow x\in A\cap A$
$\Rightarrow x\in A$ এবং $x\in A$
$\Rightarrow x\in A$ এবং $x\notin B$
$\Rightarrow x\in A $ এবং $ x\in B^\prime$
$\Rightarrow x\in A\cap B^\prime$
$\therefore A\subseteq A\cap B^\prime$
$\therefore A\cap B^\prime= A$
৩য় প্রমাণঃ
মনেকরি, 
      $x\in A\cup B^\prime$
$\Rightarrow x\in A $ অথবা $x\in B^\prime$
$\Rightarrow x\notin B$ অথবা $x\in B^\prime$
$\Rightarrow x\in B^\prime$ অথবা $x\in B^\prime$
$\Rightarrow x\in B^\prime \cup B^\prime$
$\Rightarrow x\in B^\prime$
$\therefore A\cup B^\prime \subseteq B^\prime$
মনেকরি,
      $x\in B^\prime$
$\Rightarrow x\in B^\prime \cup B^\prime$
$\Rightarrow x\in B^\prime$ অথবা $x\in B^\prime$
$\Rightarrow x\notin B$ অথবা $x\in B^\prime$
$\Rightarrow x\in A $ অথবা $x\in B^\prime$
$\Rightarrow x\in A\cup B^\prime$
$\therefore B^\prime \subseteq A\cup B^\prime$
$\therefore A\cup B^\prime =B^\prime$

সৃজনশীল প্রশ্নঃ 
$f:\mathbb{R_0}^+\rightarrow [5,\infty)$  এবং $g:[5,\infty)\rightarrow \mathbb{R}^+$ ফাংশন দুটি যথাক্রমে $f(x)=x^2+5$ এবং $g(x)=(x-5)^\tfrac{1}{2}$
(ক) $F(y+2)=3y^2-1$ হলে $F(3)=$কত?
(খ) দেখাও যে, $f$ বিপরীতযোগ্য এবং $f^{-1}=g$.
(গ) $4f^{-1}(x)=x$ হলে $x$ এর মান নির্ণয় কর ।
(ঘ) $f^{-1}$ ফাংশন কিনা নির্ধারণ কর।
(ক) নং প্রশ্নের সমাধান:
এখানে $y+2=3$
       বা, $y=3-2=1$
এখন, $F(y+2)=3y^2-1$
       বা,$F(1+2)=3×1^1-1$
       বা,$F(3)=3-1$
       $\therefore F(3)=2$
(খ) নং প্রশ্নের উত্তরঃ
যেহেতু $f:\mathbb{R_0}^+\rightarrow [5,\infty)$
সুতরাং ডোমেন, $D_f=\mathbb {R_0}^+$
কোডোমেন, $Co-D_f= [5,\infty)$
যেহেতু, $x\in \mathbb {R_0}^+$
সুতরাং $x=0$ হলে $y=f(0)=0^2+5=5$
এবং$x>0$ হলে $y>5$ হয়।
সুতরাং রেঞ্জের শর্তঃ $y\ge 5$
এই শর্ত দ্বারা গঠিত রেঞ্জ, $R_f=\left\{y\in \mathbb {R}:y\ge 5\right\}$
$=[5,\infty)$
যেহেতু $Co-D_f=R_f$
সুতরাং ফাংশনটি সার্বিক $(onto)$.
এক-এক নির্ধারণঃ 
মনেকরি, $x_1,x_2\in D_f$ এর জন্য $f(x_1)=f(x_2)$.
এখন $f(x_1)=f(x_2)$
বা,$x_1^2+5=x_2^2+5$       $\left [\because f(x)=x^2+5\right]$
বা,$x_1^2=x_2^2$
বা,$x_1=\pm x_2$      
$\therefore x_1=x_2$        $\left[\because x\ge 0\right]$
যেহেতু $f(x_1)=f(x_2)$ এর জন্য $x_1=x_2$.
সুতরাং ফাংশনটি এক-এক $(one-one)$.
আবার যেহেতু ফাংশনটি এক-এক এবং সার্বিক। তাই ফাংশনটি বিপরীতযোগ্য। অর্থাৎ, $f^{-1}$ বিদ্যমান। 
বিপরীত ফাংশন নির্ণয়ঃ 
$y=f(x)=x^2+5$
$\therefore y=f(x)$
বা,$f(x)=y$
$\therefore x=f^{-}(y)\cdots \cdots (i)$
এবং $y=x^2+5$
 বা,$x^2+5=y$
বা,$x^2=y-5$
বা,$x=\pm \sqrt{y-5}$
$\therefore x=\sqrt{y-5}\cdots (ii)$      $[\because x\in \mathbb {R}]$
$(i)$ ও $(ii)$ হতে পাই,
$x=f^{-1}(y)=\sqrt{y-4}$
বা,$y=f^{-1}(x)=\sqrt{x-5}=g(x)$
$\therefore f^{-1}=g$
(গ) নং প্রশ্নের সমাধানঃ
$4f^{-1}(x)=x$
বা,$4\sqrt{x-5}=x$
বা,$16(x-5)=x^2$
বা,$x^2=16x-80$
বা,$x^2-16x+80=0$ কে $ax^2+bx+c=0$ এর সাথে তুলনা করে পাই, $a=1,\;b=-16,\;c=80$.
$x=\dfrac{-b\pm \sqrt{b^{2}-4ac}}{2a}\\ =\dfrac{-\left( -16\right) \pm \sqrt{\left( -16\right) ^{2}-4\times 1\times 80}}{2\times 1}\\ =\dfrac{16\pm \sqrt{256-320}}{2}\\ =\dfrac{16\pm \sqrt{-64}}{2}$ ,
যা অবাস্তব সংখ্যা।
(ঘ) নং প্রশ্নের উত্তর:
$y=f^{-1}(x)=\sqrt{x-5}$
$x=6$ হলে $y=f^{-1}(6)=\sqrt{6-5}=\sqrt{1}=1$
$x$ এর একটি মানের জন্য $f^{-1}$ এর একটি মান পাওয়া যায় তাই $f^{-1}$ একটি ফাংশান।
অনুরূপ সৃজনশীল প্রশ্ন:
$F:\mathbb{R}^+\rightarrow \mathbb{R}^+$ এবং $G:\mathbb{R}^+\rightarrow\mathbb{R}^+$
 ফাংশন দুটি যথাক্রমে $F(x)=x^2$ এবং $G(x)=x^\tfrac{1}{2}$
(ক) $f(y^2+2)=3y-1$ হলে $f(11)=$কত?
(খ) দেখাও যে, $F$ বিপরীতযোগ্য এবং $F^{-1}=G$.
(গ) $4F^{-1}(x)=x$ হলে $x$ এর মান নির্ণয় কর ।
(ঘ) $F^{-1}$ ফাংশন কিনা নির্ধারণ কর।
গাণিতিক সমস্যাঃ 
$F(x)=\log\sqrt{1-x}$ এর ডোমেন ও রেঞ্জ নির্ণয় কর।
সৃজনশীল প্রশ্নঃ
$n\left( A\right) =3x-12,\; n\left( A\cap B\right) =x$ ,
 $n\left( B\right) =3x+8$ , $n\left( U\right) =25-x$ এবং $n\left(A\cup B\right)=21$
(ক) $x$ এর মান কত?
(খ) ভেনচিত্র অঙ্কন করে $x$ এর ক্ষুদ্রতম মান নির্ণয় কর।
(গ) ভেনচিত্র অঙ্কন করে $x$ এর বৃহত্তম মান নির্ণয় কর।
                   (ক) নং প্রশ্নের সমাধানঃ
আমরাজানি,
$n\left( A\cup B\right) =n\left( A\right) +n\left( B\right) -n\left( A\cap B\right) $
বা, $16=3x-12+3x+8-x$
বা, $5x-4=21$
বা, $5x=25$
 বা, $x=\dfrac{25}{5}$
$\therefore x=5$
                 (খ) নং প্রশ্নের সমাধানঃ


$x$ বা $n\left( A\cap B\right)$ ক্ষুদ্রতম যখন 
$n\left(A\cup B\right)=n\left(U\right)$
বা, $21=25-x$
বা, $x=4$
                 (গ) নং প্রশ্নের সমাধানঃ
$x$ বা $n\left( A\cap B\right)$ বৃহত্তম যখন $A\subset B$
তখন $ n\left( A\cap B\right)=n\left( A\right)$
     বা, $x=3x-12$
     বা, $-2x=-12$
     বা, $x=6$
সূচকীয় ও লগারিদমিক ফাংশন:
সৃজনশীল প্রশ্নঃ
$8y^x-y^{2x}=16……….(i)$
এবং $y=2^\tfrac{x}{2}……….(ii)$
(ক) $ii $ নং এর ডোমেন ও রেঞ্জ নির্ণয় কর।
(খ) সমীকরণ দুটির সমাধান নির্ণয় কর।
(গ) $ii$ নং এর লেখচিত্র অঙ্কন করে লেখচিত্রটির বৈশিষ্ট্য লিখ ।
                    (ক) নং প্রশ্নের সমাধানঃ
$y=f(x)=2^\tfrac{x}{2}\in \mathbb{R}$ হবে যদি $x\in \mathbb{R}$ হয়।
সুতরাং ডোমেন, $D_f=\mathbb{R}=\left(-\infty,\infty\right)$
এখন, $y=2^\tfrac{x}{2}$
    বা, $\log y=\dfrac{x}{2}\log {2}$
     বা, $2\log y=x\log {2}$
     বা, $x=\dfrac{2\log y}{\log{2}}\in \mathbb{R}$ হবে যদি $y>0$হয়।
সুতরাং রেঞ্জ, $R_f=\left\{y\in \mathbb{R}: y>0\right\}$
               '         $=\left(0,\infty\right)$
                (খ) নং প্রশ্নের সমাধানঃ
$(i)$ নং হতে পাই,
  $-y^{2x}+8y^x-16=0$
বা, $-(y^{2x}-8y^x+16)=0$
বা, $\left(y^x\right)^2-2\cdot y^x\cdot 4+4^2=0$
বা, $\left(y^x-4\right)^2=0$
বা, $y^x-4=0$
বা, $y^x=4$
বা, $2^\tfrac{x}{2}=2^2$   [$(ii)$ নং হতে]
বা, $\dfrac{x}{2}=2$
$\therefore x=4$
$x$ এর মান $(ii)$ নং সমীকরণে বসিয়ে পাই,
$y=2^{\tfrac{4}{2}}$
বা, $y=2^2$
$\therefore y=4$
অতএব নির্ণেয় সমাধান $(x,y)=(4,4)$
                           (গ) নং প্রশ্নের সমাধানঃ
$y=2^\tfrac{x}{2}$
$x$ এর কয়েকটি মানের জন্য $y$ এর সংশ্নিষ্ট মানের ছক তৈরি করিঃ

$x$ $-2$ $0$ $2$ $4$ $6$
$y$ $0.5$ $1$ $2$ $4$ $8$
$(x,y)$ $(-2,0.5)$ $(0,1)$ $(2,2)$ $(4,4)$ $(6,8)$
ছক কাগজে ক্ষুদ্রতম বর্গের বাহুর দৈর্ঘ্যকে এক একক ধরে বিন্দুগুলো স্থাপন করি। বিন্দুগুলো যোগ করলে একটি বক্ররেখা পাওয়া যায়।

লেখচিত্রের বৈশিষ্ট্যঃ
$(i)$ লেখচিত্র $y$ অক্ষকে $(0,1)$ বিন্দুতে ছেদ করে।
$(ii)$ লেখচিত্রটি একটি অপ্রতিসম বক্ররেখা।
$(iii)$ $x$ অক্ষের ঋণাত্মক দিক বক্ররেখাটির অসীমতট।
$(iv)$ $y$ অক্ষের ধনাত্মক দিকে বক্ররেখাটি ক্রমান্বয়ে বাড়তে থাকে।
$(v)$ বক্ররেখাটি হতে দেখা যায় যে, 
$x\to -\infty $ হলে $y\to 0$ এবং
$x\to \infty $ হলে $y\to \infty $ হয়।
সুতরাং $D_f=(-\infty,\infty)$
            $R_f=(0,\infty)$
সৃজনশীল প্রশ্নঃ
 $y=\log_{10}{\dfrac{5-x}{5+x}}$
(ক) ফাংশনটির ডোমেন নির্ণয় কর।
(খ) ফাংশনটির রেঞ্জ নির্ণয় কর এবং দেখাও ফাংশনটি এক-এক।
(গ) ফাংশনটির লেখচিত্র অঙ্কন করে ডোমেন ও রেঞ্জ নির্ণয় কর।
               (ক)নং প্রশ্নের সমাধানঃ
$y=f(x)=\log_{10}{\dfrac{5-x}{5+x}}\in \mathbb{R}$ হবে যদি $\dfrac{5-x}{5+x}>0$ হয়।
এখন, $\dfrac{5-x}{5+x}>0$ হবে যদি 
$5-x>0$ এবং $5+x>0$ অথবা $5-x<0$ এবং $5+x<0$,
বা, $-x>-5$ এবং $x>-5$ অথবা $-x<-5$ এবং $x<-5$,
বা, $x<5$ এবং $x>-5$ অথবা $x>5$ এবং $x<-5$.
ডোমেন, $D_f=${$x\in \mathbb{R}:x<5$ এবং $x>-5$}$\cup$ {$x\in \mathbb{R}:x>5$ এবং $x<-5$},
                $=${$x\in \mathbb{R}:-5<x<5$}$\cup \emptyset$
                $=(-5,5)$
                $=]-5,5[$
                     (খ) নং প্রশ্নের সমাধানঃ
$y=\log_{10}{\dfrac{5-x}{5+x}}$
বা, $10^y=\dfrac{5-x}{5+x}$
বা, $10^y(5+x)=5-x$
বা, $x10^y+x=5-5\cdot 10^y$
বা, $x\left(10^y+1\right)=5\left(1-10^y\right)$
বা, $x=\dfrac{5\left(1-10^y\right)}{\left(1+10^y\right)}\in \mathbb{R}$ হবে যদি $y\in\mathbb{R}$
$\therefore$ রেঞ্জ, $R_f=\mathbb{R}$
এক-এক নির্ধারণঃ 
মনেকরি, $x_1,x_2\in D_f$ এর জন্য $f(x_1)=f(x_2)$.
এখন $f(x_1)=f(x_2)$
বা,$\log_{10}{\dfrac{5-x_1}{5+x_1}}=\log_{10}{\dfrac{5-x_2}{5+x_2}}$     
বা,$\dfrac{5-x_1}{5+x_1}=\dfrac{5-x_2}{5+x_2}$
বা,$25+5x_2-5x_1-x_1x_2=25-5x_2+5x_1-x_1x_2$      
বা, $10x_2=10x_1$
$\therefore x_1=x_2$        
যেহেতু $f(x_1)=f(x_2)$ এর জন্য $x_1=x_2$.
সুতরাং ফাংশনটি এক-এক $(one-one)$.
                  (গ) নং প্রশ্নের সমাধানঃ
$y=\log_{10}{\dfrac{5-x}{5+x}}$
$x$ এর কয়েকটি মানের জন্য $y$ এর সংশ্লিষ্ট মানের ছক তৈরি করিঃ

ছক কাগজে ক্ষুদ্রতম বর্গের বাহুর দৈর্ঘ্যকে এক একক ধরে বিন্দুগুলো স্থাপন করি। বিন্দুগুলো যোগ করলে একটি বক্ররেখা পাওয়া যায়।

লেখচিত্র হতে দেখা যায় যে, 
$x\to 5$ হলে $y\to -\infty$ এবং
$x\to -5$ হলে $y\to \infty$.
সুতরাং $D_f=(-5,5)$ এবং $R_f=(-\infty,\infty)=\mathbb{R}$
সৃজনশীল প্রশ্নঃ
$f:\mathbb{R}-\left\{1\right\}→\mathbb{R}-\left\{2\right\}  ,y=f(x)=\dfrac{2x+1}{x-1}$   এবং $g(x)=lny$
 (ক) $f(x)$ এর ডোমেন ও রেঞ্জ নির্ণয় কর।
 (খ) $\dfrac{lny}{lnx}=1$ হলে xএর মান নির্ণয় কর।
 (গ) $g(x)$ এর ডোমেন ও রেঞ্জ নির্ণয় কর।
 (ঘ) $f(x)$ এবং $g(x)$ এর লেখচিত্র আঁক ।
সৃজনশীল প্রশ্নঃ
$f(x)=4^x$ এবং $g(x)=3^{x-\tfrac{1}{2}}$
(ক) $f(x)$ এর ডোমেন ও রেঞ্জ নির্ণয় কর।
(খ) $g(x)$ এর লেখচিত্র অঙ্কন করে ডোমেন ও রেঞ্জ নির্ণয় কর ।
(গ) $f(x)-g(x)=3g(x)-\dfrac{f(x)}{2}$ এর সমাধান করে $x$ এর মান নির্ণয় কর এবং দেখাও যে $g(x)$ এক-এক ফাংশন
সৃজনশীল প্রশ্নঃ
$y=\log_{k}{\tfrac{x-\sqrt{x^2-1}}{x+\sqrt{x^2+1}}}$
(ক) প্রমাণ কর যে, $y=2\log_{k}{\left(x-\sqrt{x^2-1}\right)}$.
(খ) $y=0$ হলে $x$ এর মান নির্ণয় কর।
(গ) $y$ এর ডোমেন ও রেঞ্জ নির্ণয় করে দেখাও যে ফাংশনটি এক-এক । সংকেতঃ $x\ge -\sqrt{x^2-1}\le 0$ হলে $x$ ধনাত্মক।
                    (ক) নং প্রশ্নের সমাধানঃ
$y=\log_{k}{\tfrac{x-\sqrt{x^2-1}}{x+\sqrt{x^2+1}}}$
$=\log_{k}{\tfrac{\left(x-\sqrt{x^2-1}\right)\left(x-\sqrt{x^2-1}\right)}{\left(x+\sqrt{x^2-1}\right)\left(x-\sqrt{x^2-1}\right)}}$
$=\log_{k}{\tfrac{\left(x-\sqrt{x^2-1}\right)^2}{x^2-\left(\sqrt{x^2-1}\right)^2}}$
$=\log_{k}{\tfrac{\left(x-\sqrt{x^2-1}\right)^2}{x^2-x^2+1}}$
$=\log_{k}{\left(x-\sqrt{x^2-1}\right)^2}$
$=2\log_{k}{\left(x-\sqrt{x^2-1}\right)}$
                  (খ) নং প্রশ্নের সমাধানঃ
$y=2\log_{k}{\left(x-\sqrt{x^2-1}\right)}$
বা, $0=2\log_{k}{\left(x-\sqrt{x^2-1}\right)}\cdots\cdots(i)$
বা, $2\log _{k}\left( x-\sqrt{x^{2}-1}\right) =0$
বা, $\log _{k}\left( x-\sqrt{x^{2}-1}\right) =0$
বা, $x-\sqrt{x^{2}-1}=k^{0}$
বা, $x-\sqrt{x^{2}-1}=1$
বা, $x-1=\sqrt{x^{2}-1}$
বা, $(x-1)^2=\left(\sqrt{x^{2}-1}\right)^2$
বা, $x^2-2x+1=x^2-1$
বা, $-2x=-1-1$
বা, $-2x=-2$
$\therefore x=1$
$x=1$ এর জন্য $(i)$ নং সমীকরণটি সত্য।
অতএব নির্ণেয় সমাধান $x=1$
                    (গ) নং প্রশ্নের সমাধানঃ
$y=f(x)=2\log_{k}{\left(x-\sqrt{x^2-1}\right)}\in \mathbb{R}$ হবে যদি 
  $x-\sqrt{x^2-1}>0$
বা, $x>\sqrt{x^2-1}$
বা, $x>\sqrt{x^2-1}\ge 0\cdots(i)$   [$\because$ সকল সংখ্যার বর্গমূল $\ge 0$]
বা, $\sqrt{x^2-1}\ge 0$
বা, $x^2-1\ge 0$
বা, $x^2\ge 1$
বা, $x\ge 1$     [যেহেতু $(i)$ হতে $x$ ধনাত্মক]
সুতরাং ডোমেন ,$D_f=\left\{x\in\mathbb{R}:x\ge 1\right\}$
                             $=[1,\infty)$
$x\in D_f$ হলে $y\le 0$
সুতরাং রেঞ্জ , $R_f=\left\{y\in \mathbb{R}: y\le 0\right\}$
                          $=(-\infty,0]$
                     এক-এক নির্ধারণঃ
মনেকরি, $x_1,x_2\in D_f$ এর জন্য $f(x_1)=f(x_2)$.
এখন, $f(x_1)=f(x_2)$
বা, $2\log_{k}{\left(x_1-\sqrt{x_1^2-1}\right)}=2\log_{k}{\left(x_2-\sqrt{x_2^2-1}\right)}$
বা, $x_1-\sqrt{x_1^2-1}=x_2-\sqrt{x_2^2-1}$
বা, $x_1-x_2=\sqrt{x_1^2-1}-\sqrt{x_2^2-1}$
বা, $\left(x_1-x_2\right)^2=\left(\sqrt{x_1^2-1}-\sqrt{x_2^2-1}\right)^2$
বা, $x_1^2-2x_1x_2+x_2^2=x_1^2-1$$-2\sqrt{x_1^2-1}\sqrt{x_2^2-1}+x_2^2-1$
বা, $x_1x_2=\sqrt{x_1^2x_2^2-x_1^2-x_2^2+1}+1$
বা, $\left(x_1x_2-1\right)^2=x_1^2x_2^2-x_1^2-x_2^2+1$
বা, $x_1^2x_2^2-2x_1x_2+1=x_1^2x_2^2-x_1^2-x_2^2+1$
বা, $x_1^2-2x_1x_2+x_2^2=0$
বা, $\left(x_1-x_2\right)^2=0$
বা, $x_1-x_2=0$
বা, $x_1=x_2$
যেহেতু $f(x_1)=f(x_2)$ এর জন্য $x_1=x_2$
সুতরাং ফাংশনটি এক-এক।
অনুরূপ সৃজনশীল প্রশ্নঃ
$y=\log_{k}{\tfrac{x+\sqrt{x^2-1}}{x-\sqrt{x^2-1}}}$
(ক) প্রমাণ কর যে, $y=2\log_{k}{\left(x+\sqrt{x^2-1}\right)}$.
(খ) $y=0$ হলে $x$ এর মান নির্ণয় কর।
(গ) $y$ এর ডোমেন ও রেঞ্জ নির্ণয় করে দেখাও যে ফাংশনটি এক-এক । 
 গাণিতিক সমস্যা-১:
$f(x)=\pi ^x$ এর লেখচিত্র অঙ্কন করে ডোমেন ও রেঞ্জ নির্ণয় কর।
গাণিতিক সমস্যা-২:
$y=f(x)=\log_e \dfrac{4-x}{4+x}$ এর লেখচিত্র অঙ্কন করে ডোমেন ও রেঞ্জ নির্ণয় কর।
গাণিতিক সমস্যা-৩:
$y=f(x)=\log_{10}\dfrac{4+x}{x-4}$ এর ডোমেন ও রেঞ্জ নির্ণয় কর।
গাণিতিক সমস্যাঃ
লেখচিত্র অঙ্কন করে দেখাও যে, $y=2^x$ ফাংশন এবং তার বিপরীত ফাংশনের লেখ প্রতিসম।
গাণিতিক সমস্যাঃ
$f(3x)=x+2$ হলে $f(3-3x)$ কে $x$ চলকের ফাংশনরূপে প্রকাশ কর।
সমাধানঃ
     $f(3x)=x+2$
বা, $f(3x)=\dfrac{3x}{3}+2$
বা, $f(a)=\dfrac{a}{3}+2$                   ধরি, $3x=a$
$\therefore f(3-3x)=\dfrac{3-3x}{3}+2$
                             $=\dfrac{3-3x+6}{3}$
                             $=\dfrac{9-3x}{3}$
                             $=\dfrac{3(3-x)}{3}$
                             $=3-x$



                      বহুনির্বাচনী প্রশ্নঃ 
                        মডেল টেস্ট-১
পূর্ণমান-২৫                                       সময়-২৫ মিনিট
১.$F(x)=\dfrac{1}{\sqrt{1-x}}$ ফাংশনের ডোমেন কত?
(ক) $\left\{x\in \mathbb{R}:x\ne 1\right \}$    (খ) $\left\{x\in \mathbb{R}:x\ge 1\right \}$
(গ) $\left\{x\in \mathbb{R}:x<1\right \}$   (ঘ) $\left\{x\in \mathbb{R}:x>1\right \}$
২. $y=\dfrac{x-1}{x+1}$ এর লেখচিত্র 
  $i.$ বক্ররেখা 
  $ii.$ বিচ্ছিন্ন 
  $iii.$ অবিচ্ছিন্ন 
কোনটি সঠিক?
  (ক) $i,ii$      (খ) $i,iii$     (গ) $ii, iii$      (ঘ) $i,ii,iii$
৩.কোন সেটের শক্তি সেটের উপাদান সংখ্যা $\log_{\sqrt{3}}3$ হলে  ঐ সেটের উপাদান সংখ্যা কত?
  (ক) $3$          (খ) $2$          (গ) $1$             (ঘ) $0$
৫. $y=\sqrt[3]{x}$  এর ডোমেন কত?
   (ক)  [0,∞)                (খ) (0,∞) 
   (গ) (-∞,0)              (ঘ) (-∞,∞)
৬. $f(x)=\dfrac{|x|}{x}  ,x≠0$ ফাংশনটির রেঞ্জ কত?
   (ক) $(-1,1)$                         (খ) $\left\{-1,1\right\}$      
   (গ) $(-1,1]$                         (ঘ) $[-1,1]$
৭. মূলদ সংখ্যার সেট $Q=$
$(i). ${$x:x=\dfrac{p}{q}$ এবং $p,q\in \mathbb{Z}$}
$(ii).${$x:x=\dfrac{p}{q}$ এবং $p\in\mathbb{Z},q\in\mathbb{N}$}
$(iii).${$x:x=\dfrac{p}{q}$ এবং $p\in\mathbb{N},q\in\mathbb{Z},q\ne 0$}
কোনটি সঠিক?
(ক) $i,ii$    (খ) $i,iii$    (গ) $ii,iii$    (ঘ) $i,ii,iii$
৮. $A,\;B,\;C$ সেটের ক্ষেত্রে -
$i. A\cap \left( B\cup C\right) =\left( A\cap B\right) \cup\left( A\cap C\right)$
$ii.A\backslash \left( B\cap C\right) =\left(A\backslash B\right) \cap \left( A\backslash C\right) $
$iii.A\times \left( B\cup C\right) =\left( A\times B\right) \cup\left( A\times C\right)$
কোনটি সঠিক?
(ক) $i,ii$    (খ) $i,iii$    (গ) $ii,iii$    (ঘ) $i,ii,iii$
১০. $x$ এর সর্বোচ্চ মান কত?
  (ক) $2$         (খ) $3$          (গ) $4$             (ঘ) $5$
১১. $x$ এর সর্বনিম্ন মান কত?
  (ক) $1$         (খ) $2$          (গ) $3$             (ঘ) $4$
উত্তরপত্রঃ 
১.(গ)
২১. $\mathrm {64x^{3}\div 27a^{-3})^{-2/3}=?}$ 
(a) $\mathrm {\dfrac{9}{16} \dfrac{1}{x^2a^2}}$ 
(b) $\mathrm{\dfrac{3}{4} x^{-2}a^{-2}}$
(c)$\mathrm{\dfrac{9}{16ax}}$
পরবর্তী পোস্ট পূর্ববর্তী পোস্ট
NO COMMENT
Add Comment

Enter Comment

comment url